LSAT and Law School Admissions Forum

Get expert LSAT preparation and law school admissions advice from PowerScore Test Preparation.

 Administrator
PowerScore Staff
  • PowerScore Staff
  • Posts: 8917
  • Joined: Feb 02, 2011
|
#26034
Complete Question Explanation
(The complete setup for this game can be found here: lsat/viewtopic.php?t=10807)

The correct answer choice is (E)

If aisle 2 contains only S, this would force H to aisle 1, and P—to aisle 3. Additionally, to satisfy the F > M sequence, F must be added to aisle 1 and M—to aisle 3:
oct12_game_3_#17_diagram_1.png
As in question #13, the only uncertainty here is the placement of R. Since R must be located on the same aisle as either F or M, it must be placed in either aisle 1 or aisle 3:
oct12_game_3_#17_diagram_2.png
In a Must Be True, Except question such as this one, you should immediately seek answer choices that contain R, because the truth value for the remaining five variables is “must be true.” Only answer choice (E) qualifies. Since R could be located in either aisles 1 or 3, answer choice (E) does not need to be true, and is therefore the correct answer choice to this Must Be True, Except question.

Get the most out of your LSAT Prep Plus subscription.

Analyze and track your performance with our Testing and Analytics Package.